Could somebody explain this?

  • MHB
  • Thread starter stripedcat
  • Start date
  • Tags
    Explain
In summary: Your first question is how to solve the 'initial value problem'. You can solve the initial value problem by setting the initial condition equal to the desired solution and solving for the variables. $\displaystyle \frac{d p}{d h} = k\ p,\ p(0)= 30\ $$\displaystyle \frac{d p}{p} = k\ d h\ $ $\displaystyle \int \frac{d p}{p} = k\ \int d h \implies \ln p = k\ h + \ln c \implies p = c\ e^{k\ h}\ $$\displaystyle \
  • #1
stripedcat
44
0
I have the solution so to speak, but I'd like some explanation for it. You do not have to type out the explanation yourself if you have a link to something that might shed insight. I don't know all of the format commands for the math yet, so forgive me for the formatting issues that are bound to crop up.

Okay, here we go!

Under ideal conditions, air pressure decreases continuously with the height above sea level at a rate proportional to the pressure at that height. The barometer reads 30 inches at sea level and 15 inches at 18,000 feet. Find the barometric pressure at 35,000 feet.

dp/dh = kp, P(0)=30

\(\displaystyle P(h) = 30e^{kh}\)

First question is, how are we getting that second equation there? I see where the number and variables come from but I'm not quite sure how they were arranged that way.

Our first h value is 18,000

\(\displaystyle P(18,000) = 30e^{18,000k}\)

As stated in the problem, this will be = 15, right? Not sure if that's important, just wanted to verify.

k = ln(1/2)/18,000

I don't know where they got that ln (1/2) from though I do know it's going to become -ln(2)/18,000

I could continue the problem, but that last bit there is what I really need answered. I have knowledge gaps in my math, so I'm sure I'm missing something.
 
Physics news on Phys.org
  • #2
stripedcat said:
I have the solution so to speak, but I'd like some explanation for it. You do not have to type out the explanation yourself if you have a link to something that might shed insight. I don't know all of the format commands for the math yet, so forgive me for the formatting issues that are bound to crop up.

Okay, here we go!

Under ideal conditions, air pressure decreases continuously with the height above sea level at a rate proportional to the pressure at that height. The barometer reads 30 inches at sea level and 15 inches at 18,000 feet. Find the barometric pressure at 35,000 feet.

dp/dh = kp, P(0)=30

\(\displaystyle P(h) = 30e^{kh}\)

First question is, how are we getting that second equation there? I see where the number and variables come from but I'm not quite sure how they were arranged that way.

Our first h value is 18,000

\(\displaystyle P(18,000) = 30e^{18,000k}\)

As stated in the problem, this will be = 15, right? Not sure if that's important, just wanted to verify.

k = ln(1/2)/18,000

I don't know where they got that ln (1/2) from though I do know it's going to become -ln(2)/18,000

I could continue the problem, but that last bit there is what I really need answered. I have knowledge gaps in my math, so I'm sure I'm missing something.

Wellcome on MHB stripedcat!...

Your first question is how to solve the 'initial value problem'...

$\displaystyle \frac{d p}{d h} = k\ p,\ p(0)= 30\ (1)$

The equation in (1) is the most simple example of ODE, in which the variables are separable so that You can write...

$\displaystyle \frac{d p}{p} = k\ d h\ (2)$

... and integrating both terms of (2) You obtain...

$\displaystyle \int \frac{d p}{p} = k\ \int d h \implies \ln p = k\ h + \ln c \implies p = c\ e^{k\ h}\ (3)$

The next step is to find the constant c and k in (3). Are You able to do that?...

Kind regards

$\chi$ $\sigma$
 
  • #3
Thank you for the welcome!

$\displaystyle \frac{d p}{d h} = k\ p,\ p(0)= 30\ $

$\displaystyle \frac{d p}{p} = k\ d h\ $

Alright so we arrange it as you have done, multiply both sides by dh and divide both sides by p. No problem. I know how dp/p became ln |p|, p^-1 dp = ln p

Now for the right side: k dh, that would become k(h+c) wouldn't it? Thus making it ln p = k(h+c)

Both sides by e maing it p = e^k(h+C), I know to bring down that C to make it p = Ce^kh

Okay, got that now! Thanks.

Constant? 30 e? I'm not sure what you're asking but to press on with what I think occurs next.. I did figure out the ln ½ bit

ln(30e^18000k) =18000k + ln(30)
ln(15) = 18000k + ln(30)

Play the number flipping game to isolate k, ln(15) – ln(30) = 18000k, which becomes (ln(15) – ln(30))/18000 = k

ln(15)-ln(30) = 1/2ln = -ln(2) so -ln(2)/18000 = k

Now that we have k...

\(\displaystyle P(h) = 30e^{(-ln(2)/18000)h}\)

Is that right? Plugging in the 35000 for h gives 7.8~
 
  • #4
stripedcat said:
Now for the right side: k dh, that would become k(h+c) wouldn't it? Thus making it ln p = k(h+c)

It's better to say \(\displaystyle kh + C\) as you've written \(\displaystyle kh+kc\). It's ok this time because you get to say \(\displaystyle C' = kc\)

Both sides by e maing it p = e^k(h+C), I know to bring down that C to make it p = Ce^kh

You have \(\displaystyle p = e^{k(h+c)} = e^{kh+kc} = e^{kc}e^{kh}\). However, \(\displaystyle p = Ce^{kh}\) is right.
Constant? 30 e? I'm not sure what you're asking but to press on with what I think occurs next.. I did figure out the ln ½ bit

\(\displaystyle 30e\) is a constant but chisigma is asking you to find the values of \(\displaystyle c\) and \(\displaystyle k\) using the information given
ln(30e^18000k) =18000k + ln(30)
ln(15) = 18000k + ln(30)

Play the number flipping game to isolate k, ln(15) – ln(30) = 18000k, which becomes (ln(15) – ln(30))/18000 = k

ln(15)-ln(30) = 1/2ln = -ln(2) so -ln(2)/18000 = k

I'm not sure what you're doing here but that first line is incorrect - logarithms do not work that way. Because you have an exponent in the logarithm you need to use the power law: \(\displaystyle \ln(a^b) = b\ln(a)\). In your case you end up with \(\displaystyle \ln(30e^{18000k}) = 18000k\ln(30)\).

edit: I see what you did now, that works

\(\displaystyle 15 = 30e^{18000k}\)

Divide both sides by 30 (this is where you'll come across the 1/2 as in the OP)

\(\displaystyle \frac{1}{2} = e^{18000k}\)

\(\displaystyle \ln \left(\dfrac{1}{2}\right) = 18000k\)

\(\displaystyle k = \dfrac{\ln \left(\dfrac{1}{2}\right)}{18000}\)
 

1. What is the purpose of "Could somebody explain this?"

The phrase "Could somebody explain this?" is a request for someone to provide a clear and concise explanation of a concept or topic that the person asking does not fully understand.

2. How can I effectively ask someone to explain something to me?

To effectively ask someone to explain something to you, it is important to be specific about what you are confused about and to approach the person politely and respectfully. Additionally, providing some background information about your current understanding of the topic can help the person tailor their explanation to your level of knowledge.

3. Is it okay to ask for clarification if I don't understand something?

Absolutely! Asking for clarification is a normal and important part of the learning process. It shows that you are actively trying to understand and engage with the information being presented.

4. How can I make sure I understand the explanation given to me?

One way to make sure you understand an explanation is to ask follow-up questions or paraphrase what was said to confirm your understanding. Additionally, you can take notes or try to explain the concept to someone else to solidify your understanding.

5. What if I still don't understand even after someone has explained it to me?

If you still don't understand after someone has explained something to you, don't be afraid to ask for further clarification or seek out additional resources on the topic. Everyone learns and processes information differently, so it's okay if it takes a few attempts for something to click for you.

Similar threads

Replies
3
Views
1K
  • Introductory Physics Homework Help
Replies
2
Views
898
  • Advanced Physics Homework Help
Replies
1
Views
1K
  • Mechanical Engineering
Replies
11
Views
2K
Replies
1
Views
581
  • Introductory Physics Homework Help
Replies
8
Views
2K
  • Mechanical Engineering
Replies
15
Views
808
  • Introductory Physics Homework Help
Replies
9
Views
1K
  • Introductory Physics Homework Help
Replies
1
Views
901
Back
Top